4 votos

Si $X \times X$ es normal, entonces es $X \times X \times X$ ¿Normal?

Estoy estudiando un poco de teoría de la dimensión topológica para los espacios producto, y al tratar de construir un cierto tipo de contraejemplo se ha vuelto relevante considerar la pregunta del título anterior. Estoy interesado en encontrar un espacio normal $X$ cuyo producto consigo mismo es finalmente no normal, pero no inmediatamente.

En realidad, para mi aplicación no es importante que ocurra en tres pasos en lugar de más. Una pregunta alternativa sería: ¿Existe un espacio normal $X$ con $X \times X = Y$ normal, pero $Y \times Y$ ¿no es normal?

El problema original está aquí:

https://mathoverflow.net/questions/315657/if-textdimx-times-x-2-textdimx-does-textdimxn-n-textdim

Gracias por cualquier ayuda.

Como se menciona en un comentario más abajo, si asumimos que $X$ es un espacio Hausdorff compacto y que $X \times X \times X$ es completamente normal, entonces $X$ es metrizable. Por tanto, es lógico que un contraejemplo compacto sea más difícil (si no imposible) de construir. El autor del artículo enlazado se pregunta en voz alta si la normalidad completa de $X \times X$ es suficiente para la metrizabilidad de $X$ por lo que también puede ser aconsejable evitar los casos en los que $X \times X$ es completamente normal.

1 votos

Esto no es una respuesta, pero de alguna manera está relacionado: math.stackexchange.com/q/2872102

0 votos

Interesante resultado, lo mencionaré en una edición.

4voto

Enoch the Red Puntos 2197

Se puede encontrar una construcción en (o destetar de)

donde se demuestra el siguiente resultado notable:

Teorema 1. Por cada $k$ y $m$ tal que $1 \leq k \leq m \leq \omega$ existe un espacio separable y primero coutnable $X = X(k,m)$ tal que

  1. $X^n$ es paracompacto (Lindelöf, subparacompacto) si y sólo si $n < k$ ,
  2. $X^n$ es normal (normal a nivel de colección) si y sólo si $n < m$ .

En particular, podemos construir un espacio (normal) tal que el fallo de la normalidad de sus potencias ocurre primero en cualquier potencia finita prescrita.

0 votos

¡Vaya, qué resultado!

i-Ciencias.com

I-Ciencias es una comunidad de estudiantes y amantes de la ciencia en la que puedes resolver tus problemas y dudas.
Puedes consultar las preguntas de otros usuarios, hacer tus propias preguntas o resolver las de los demás.

Powered by:

X